Proving (n+1)^4 < 4n^4 for n >= 3

  • Thread starter Thread starter kmeado07
  • Start date Start date
  • Tags Tags
    Induction Proof
Click For Summary
To prove that (n+1)^4 < 4n^4 for n >= 3, the method of mathematical induction is suggested. The base case for n=3 should be verified first. Assuming the inequality holds for n, the next step is to show it holds for n+1, transforming the expression into (n+2)^4 < 4(n+1)^4. The discussion emphasizes that since the question requires a strict inequality, additional positive terms can be added to the left side without violating the inequality. The importance of ensuring that any subtracted terms are positive is also highlighted to maintain the validity of the inequality.
kmeado07
Messages
40
Reaction score
0

Homework Statement


Show that (n+1)^4 < 4n^4 whenever n >= 3


Homework Equations





The Attempt at a Solution



I need to prove this by induction, so i assume it is true and then prove that when n=n+1 it is also true.

so it would become (n+2)^4 < 4(n+1)^4

Im not sure how to continue this though.
 
Physics news on Phys.org
The first step is to check it for n = 3. Then assume it is true for n. I haven't tried this, but I would start by something like:
((n + 1) + 1)^4 = (n + 1)^4 + 4 (n + 1)^3 + ...
Then use the induction hypothesis.

Note that, because your question is "show that ... is STRICTLY SMALLER than ... " you can always throw away terms you know are positive. For example, for n >= 3, if you have that (n + 1)^4 < 4 n^4 then you also have that (n + 1)^4 + 14n + 3 < 4 n^4, because you are only adding terms which make the left hand side bigger so the inequality will keep holding. Watch out however, that the terms have to be positive, if (n + 1)^4 < 4 n^4 then it needn't be true that (n + 1)^4 - 20 < 4 n^4.
 
Question: A clock's minute hand has length 4 and its hour hand has length 3. What is the distance between the tips at the moment when it is increasing most rapidly?(Putnam Exam Question) Answer: Making assumption that both the hands moves at constant angular velocities, the answer is ## \sqrt{7} .## But don't you think this assumption is somewhat doubtful and wrong?

Similar threads

  • · Replies 9 ·
Replies
9
Views
3K
Replies
6
Views
2K
  • · Replies 3 ·
Replies
3
Views
2K
  • · Replies 7 ·
Replies
7
Views
2K
  • · Replies 8 ·
Replies
8
Views
1K
  • · Replies 4 ·
Replies
4
Views
1K
  • · Replies 7 ·
Replies
7
Views
1K
  • · Replies 1 ·
Replies
1
Views
1K
  • · Replies 1 ·
Replies
1
Views
2K
  • · Replies 13 ·
Replies
13
Views
2K